Is this correct??

\(\large{I=\displaystyle \int^{\infty}_{0}\frac{\cos^{2} x}{x^2}.dx=\infty}\).

I approached it as

=01sin2xx2.dx\large{= \displaystyle \int^{\infty}_{0}\frac{1- \sin^{2} x}{x^2}.dx}

=0sin2xx2.dx\large{= \infty-\displaystyle \int^{\infty}_{0}\frac{\sin^{2} x}{x^2}.dx}

=π2\large{= \infty-\frac{\pi}{2}}

== \infty

#Calculus

Note by Tanishq Varshney
6 years ago

No vote yet
1 vote

  Easy Math Editor

This discussion board is a place to discuss our Daily Challenges and the math and science related to those challenges. Explanations are more than just a solution — they should explain the steps and thinking strategies that you used to obtain the solution. Comments should further the discussion of math and science.

When posting on Brilliant:

  • Use the emojis to react to an explanation, whether you're congratulating a job well done , or just really confused .
  • Ask specific questions about the challenge or the steps in somebody's explanation. Well-posed questions can add a lot to the discussion, but posting "I don't understand!" doesn't help anyone.
  • Try to contribute something new to the discussion, whether it is an extension, generalization or other idea related to the challenge.
  • Stay on topic — we're all here to learn more about math and science, not to hear about your favorite get-rich-quick scheme or current world events.

MarkdownAppears as
*italics* or _italics_ italics
**bold** or __bold__ bold

- bulleted
- list

  • bulleted
  • list

1. numbered
2. list

  1. numbered
  2. list
Note: you must add a full line of space before and after lists for them to show up correctly
paragraph 1

paragraph 2

paragraph 1

paragraph 2

[example link](https://brilliant.org)example link
> This is a quote
This is a quote
    # I indented these lines
    # 4 spaces, and now they show
    # up as a code block.

    print "hello world"
# I indented these lines
# 4 spaces, and now they show
# up as a code block.

print "hello world"
MathAppears as
Remember to wrap math in \( ... \) or \[ ... \] to ensure proper formatting.
2 \times 3 2×3 2 \times 3
2^{34} 234 2^{34}
a_{i-1} ai1 a_{i-1}
\frac{2}{3} 23 \frac{2}{3}
\sqrt{2} 2 \sqrt{2}
\sum_{i=1}^3 i=13 \sum_{i=1}^3
\sin \theta sinθ \sin \theta
\boxed{123} 123 \boxed{123}

Comments

For the original question: The reasoning is almost correct. The only drawback is using \infty as a real number. Assuming 0cos2(x)x2dx\int_0^\infty\frac{\cos^2(x)}{x^2}\,\mathrm{d}x converged, then 0cos2(x)x2dx+0sin2(x)x2dxconvergent=01x2dx\int_0^\infty\frac{\cos^2(x)}{x^2}\,\mathrm{d}x+\underbrace{\int_0^\infty\frac{\sin^2(x)}{x^2}\,\mathrm{d}x}_{\text{convergent}}=\int_0^\infty\frac1{x^2}\,\mathrm{d}x would also converge. However, we know that 01x2dx\int_0^\infty \dfrac{1}{x^2}dx diverges hence the original integral is divergent.

User 123 - 6 years ago

Log in to reply

Is this fine? @Azhaghu Roopesh M

User 123 - 6 years ago

Log in to reply

I guess so . The integral clearly diverges . Plus if one was bored and has some time at hand ,then he can even do an IBP and apply the limits to get the answer .

I can't calculate the integral right now , since I don't have anything to write upon (I'm at a party :P) , but I'll check it once I reach home .

Log in to reply

@A Former Brilliant Member Integral will not converge irrespective of upper limit(except 0).

Krishna Sharma - 6 years ago

I'm fascinated by this problem, so here is an alternative method before I retire to bed.

0cos2xx2dx>0π/4cos2xx2dx120π/4dxx2=\int_0^\infty\frac{\cos^2x}{x^2}dx>\int_0^{\pi/4}\frac{\cos^2x}{x^2}dx\ge\frac12\int_0^{\pi/4}\frac{dx}{x^2}=\infty

A small explanation:

  • First Inequality: Since cos2(x)/(x2)cos^2(x)/(x^2) is positive, the integral is greater than the integral of the same function over a smaller interval.

  • Second Inequality: for x[0,π/4],cos2(x)1/2x∈[0,π/4], cos^2(x)≥1/2.

  • Third Inequality: the integral of xnx^{-n} over [0,k][0,k] diverges for n1n≥1 and k>0k>0.

Hope this helps for now.

User 123 - 6 years ago

Log in to reply

Sorry, I've to go to bed right now (have FIITJEE tomorrow from 10:30 - 5:30). Will surely respond in detail when I get back tomorrow,

User 123 - 6 years ago

Log in to reply

ok , thats the point, any way then how to deal with it, i'll wait for tomorrow

Tanishq Varshney - 6 years ago

Log in to reply

@Tanishq Varshney Sorry, I have to just confirm once. Until then, I'll edit my last comment as it could perhaps be wrong.

User 123 - 6 years ago

Tanishq, your method and answer should be correct. I am saying so, because Wolfram Alpha is also showing the same result. I also tried it using Integration By Parts but got stuck in the midway.

You can also check out these links, though these are of (sinx)^2/(x^2). Maybe of little use to you ...... As, you've already shown that in your post.

http://math.stackexchange.com/questions/13344/proof-of-int-0-infty-left-frac-sin-xx-right2-mathrm-dx-frac-pi2 http://math.stackexchange.com/questions/141695/how-to-calculate-the-integral-of-sin2x-x2 http://math.stackexchange.com/questions/13344/proof-of-int-0-infty-left-frac-sin-xx-right2-mathrm-dx-frac-pi2?lq=1

Rubayet Tusher - 6 years ago

Log in to reply

@Rubayet Tusher While Wolfram is indeed a very useful tool, you should not base your results on what it shows. On occasion, I have seen Wolfram mark convergent integrals as divergent. It often marks double limits as divergent too (in fact. in one of his problems, @Brian Charlesworth Sir has mentioned this). Brian Sir, I am truly sorry for having @mentioned you. I was typing your name and had unknowingly pressed the 'enter' key, leading to a Notification being sent to you. I'm truly sorry for the inconvenience caused, Sir.

User 123 - 6 years ago

Log in to reply

Ishan Dasgupta Samarendra I'm extremely sorry for my that comment. Infact, I was not aware of that fact about Wolfram Alpha. Thanks for informing that. And, is it possible to prove using Integration By Parts? I tried it but couldn't go to the end. If it is possible using IBP, please post it when you get time. Thanks again for pointing out my fault.

Rubayet Tusher - 6 years ago

Log in to reply

@Rubayet Tusher Hello Rubayet.Glad to be of help:) And there's absolutely no need to be sorry! Come on, all of us are allowed to make misconceptions! I don't know if it will be possible through IBP and am too tired to think now (sorry but I've got a splitting headache). I'll surely try it when I get home late evening tomorrow (I have FIITJEE throughout tomorrow). Perhaps you could @mention Calvin Sir. He would definitely help you out!

User 123 - 6 years ago

Log in to reply

@User 123 Ok, Ishan. No Problem. You may post it whenever you get time.

Rubayet Tusher - 6 years ago

Log in to reply

@Rubayet Tusher @Rubayet Tusher Hello Rubayet. I'm really sorry for the delay. The last few days have been quite hectic and I had quite forgotten about this. My apologies once again to you. See, this probably cannot be done using IBP and I'll try to explain why. First here is one general rule which often comes useful in general cases:

  • a1xpdx\int_a^\infty \dfrac{1}{x^p} dx for a>0a>0 diverges for p1p\le1.

Now, assuming we try to Integrate by Parts, we would get a first term which we would have to evaluate and a second term (namely the Integral). Thus, if the original Integral was divergent (and in this case it is), then at least one of the terms must be equal to \infty. However, this would again result in \infty being used as a real number (the way Tanishq had done), which is incorrect. Consequently, we cannot use Integration by parts for this Integral.

User 123 - 6 years ago

Log in to reply

@User 123 @Ishan Dasgupta Samarendra No need to apologize or sorry. You should be busy with your JEE Preparation at present. So, no problem for the delay. Thanks for posting it and informing me about this fact. Thanks again.

Rubayet Tusher - 6 years ago

@Rubayet Tusher @Rubayet Tusher Note that if we have abf(x)dx\int_a^b f(x)dx such that the integral diverges at either aa or bb, then we can express the limit at which abf(x)dx\int_a^b f(x)dx diverges as tt. For our example, let us assume that abf(x)dx\int_a^b f(x)dx diverges at bb. Assuming we can express atf(x)dx\int_a^t f(x)dx in a closed form (after performing whatever technique of Integration you use) which does not contain another Integral, then we can take the limit of tt to bb from the left (since for the entire interval of Integration except at bb the integral is convergent) and if the limit exists and is finite, then the Integral is convergent. This can be extended to cases where aa or both the limits of the interval of Integration is a point where the integral diverges.

User 123 - 6 years ago
×

Problem Loading...

Note Loading...

Set Loading...